Are there continuous functions who are the same in an interval but differ in at least one other point? Announcing the arrival of Valued Associate #679: Cesar Manara Planned maintenance scheduled April 17/18, 2019 at 00:00UTC (8:00pm US/Eastern)Function which is continuous everywhere in its domain, but differentiable only at one pointAre there non-periodic continuous functions with this property?Derivative defined at some point but not continuous there?Are the two statements about continuous functions equivalent?Prove or disprove: for any two given functions, one must be upper bounding the otherIf a function is derivable in a point then there exists an open interval around the point in which the function is continuousIs there a function on a compact interval that is differentiable but not Lipschitz continuous?Are there continuous functions for which the epsilon-delta property doesn't hold?Show that two continuous functions that are surjective over the same interval intersectProve a non-constant continuous function on a compact interval must admit at least one non-local extremum

Strange behaviour of Check

Stop battery usage [Ubuntu 18]

Biased dice probability question

Geometric mean and geometric standard deviation

Stars Make Stars

Problem when applying foreach loop

Windows 10: How to Lock (not sleep) laptop on lid close?

How should I respond to a player wanting to catch a sword between their hands?

What do you call the holes in a flute?

Was credit for the black hole image misattributed?

How to say that you spent the night with someone, you were only sleeping and nothing else?

Single author papers against my advisor's will?

How is simplicity better than precision and clarity in prose?

What LEGO pieces have "real-world" functionality?

Estimate capacitor parameters

Replacing HDD with SSD; what about non-APFS/APFS?

Direct Experience of Meditation

What are the performance impacts of 'functional' Rust?

What's the point in a preamp?

Unable to start mainnet node docker container

Notation for two qubit composite product state

What is the electric potential inside a point charge?

Area of a 2D convex hull

Jazz greats knew nothing of modes. Why are they used to improvise on standards?



Are there continuous functions who are the same in an interval but differ in at least one other point?



Announcing the arrival of Valued Associate #679: Cesar Manara
Planned maintenance scheduled April 17/18, 2019 at 00:00UTC (8:00pm US/Eastern)Function which is continuous everywhere in its domain, but differentiable only at one pointAre there non-periodic continuous functions with this property?Derivative defined at some point but not continuous there?Are the two statements about continuous functions equivalent?Prove or disprove: for any two given functions, one must be upper bounding the otherIf a function is derivable in a point then there exists an open interval around the point in which the function is continuousIs there a function on a compact interval that is differentiable but not Lipschitz continuous?Are there continuous functions for which the epsilon-delta property doesn't hold?Show that two continuous functions that are surjective over the same interval intersectProve a non-constant continuous function on a compact interval must admit at least one non-local extremum










10












$begingroup$


You are given a function $f: mathbbRrightarrow mathbbR$. Every derivative $fracd^ndx^n(f(x)), ,n >0$ of the function is continuous.



Is there a function $g: mathbbRrightarrow mathbbR$, for which every derivative $fracd^ndx^n(g(x)), ,n >0$ is also continuous, such that:
$$forall xin[a,b]: , g(x) = f(x)land , exists x notin [a,b]: f(x) neq g(x),, a neq b$$



Thanks!










share|cite|improve this question











$endgroup$







  • 2




    $begingroup$
    FYI, this can't happen with complex-differentiable $f: mathbbC to mathbbC$. Such functions are immediately infinitely often differentiable, so it indeed matches your setting. If two complex-differentiable functions agree on an open set, they already agree everywhere on their domain.
    $endgroup$
    – ComFreek
    yesterday











  • $begingroup$
    How to proof that?
    $endgroup$
    – TVSuchty
    yesterday















10












$begingroup$


You are given a function $f: mathbbRrightarrow mathbbR$. Every derivative $fracd^ndx^n(f(x)), ,n >0$ of the function is continuous.



Is there a function $g: mathbbRrightarrow mathbbR$, for which every derivative $fracd^ndx^n(g(x)), ,n >0$ is also continuous, such that:
$$forall xin[a,b]: , g(x) = f(x)land , exists x notin [a,b]: f(x) neq g(x),, a neq b$$



Thanks!










share|cite|improve this question











$endgroup$







  • 2




    $begingroup$
    FYI, this can't happen with complex-differentiable $f: mathbbC to mathbbC$. Such functions are immediately infinitely often differentiable, so it indeed matches your setting. If two complex-differentiable functions agree on an open set, they already agree everywhere on their domain.
    $endgroup$
    – ComFreek
    yesterday











  • $begingroup$
    How to proof that?
    $endgroup$
    – TVSuchty
    yesterday













10












10








10


1



$begingroup$


You are given a function $f: mathbbRrightarrow mathbbR$. Every derivative $fracd^ndx^n(f(x)), ,n >0$ of the function is continuous.



Is there a function $g: mathbbRrightarrow mathbbR$, for which every derivative $fracd^ndx^n(g(x)), ,n >0$ is also continuous, such that:
$$forall xin[a,b]: , g(x) = f(x)land , exists x notin [a,b]: f(x) neq g(x),, a neq b$$



Thanks!










share|cite|improve this question











$endgroup$




You are given a function $f: mathbbRrightarrow mathbbR$. Every derivative $fracd^ndx^n(f(x)), ,n >0$ of the function is continuous.



Is there a function $g: mathbbRrightarrow mathbbR$, for which every derivative $fracd^ndx^n(g(x)), ,n >0$ is also continuous, such that:
$$forall xin[a,b]: , g(x) = f(x)land , exists x notin [a,b]: f(x) neq g(x),, a neq b$$



Thanks!







real-analysis calculus






share|cite|improve this question















share|cite|improve this question













share|cite|improve this question




share|cite|improve this question








edited 2 days ago









ZeroXLR

1,657620




1,657620










asked 2 days ago









TVSuchtyTVSuchty

856




856







  • 2




    $begingroup$
    FYI, this can't happen with complex-differentiable $f: mathbbC to mathbbC$. Such functions are immediately infinitely often differentiable, so it indeed matches your setting. If two complex-differentiable functions agree on an open set, they already agree everywhere on their domain.
    $endgroup$
    – ComFreek
    yesterday











  • $begingroup$
    How to proof that?
    $endgroup$
    – TVSuchty
    yesterday












  • 2




    $begingroup$
    FYI, this can't happen with complex-differentiable $f: mathbbC to mathbbC$. Such functions are immediately infinitely often differentiable, so it indeed matches your setting. If two complex-differentiable functions agree on an open set, they already agree everywhere on their domain.
    $endgroup$
    – ComFreek
    yesterday











  • $begingroup$
    How to proof that?
    $endgroup$
    – TVSuchty
    yesterday







2




2




$begingroup$
FYI, this can't happen with complex-differentiable $f: mathbbC to mathbbC$. Such functions are immediately infinitely often differentiable, so it indeed matches your setting. If two complex-differentiable functions agree on an open set, they already agree everywhere on their domain.
$endgroup$
– ComFreek
yesterday





$begingroup$
FYI, this can't happen with complex-differentiable $f: mathbbC to mathbbC$. Such functions are immediately infinitely often differentiable, so it indeed matches your setting. If two complex-differentiable functions agree on an open set, they already agree everywhere on their domain.
$endgroup$
– ComFreek
yesterday













$begingroup$
How to proof that?
$endgroup$
– TVSuchty
yesterday




$begingroup$
How to proof that?
$endgroup$
– TVSuchty
yesterday










2 Answers
2






active

oldest

votes


















16












$begingroup$

Define the real functions $f$ and $g$ thus:
$$
f(x) = begincases expBig(-frac1(x - 1)^2Big) &textif x > 1 \
0 &textif x in [-1, 1] \
expBig(-frac1(x + 1)^2Big) &textif x < -1
endcases
$$
and
$g(x) = 0$. $f$ and $g$ are both $0$ on $[-1, 1]$ but they differ in value everywhere else.



Obviously $g$ is continuously differentiable infinitely many times as it is a constant function. You can also check that $f$ is continuously differentiable infinitely many times at $x = -1$ and $x = 1$ by applying L'Hôpital's rule inductively (Edit: A more elementary demonstration is given below based on the exponential series). Checking this is a fine exercise in Real Analysis; you should try it. Here is a first taste of it:
beginalign*
lim_x to 1^+fracdf(x)dx &= limlimits_x to 1^+frac2expbig(- frac1(x - 1)^2big)(x - 1)^3 \
&= 2lim_x to 1^+fracfrac1(x - 1)^3expbig(frac1(x - 1)^2big) quadtextthis limit is of the form fracinftyinfty text so L'Hôpital applies \
&= 2 lim_x to 1^+fracfracddx(x - 1)^-3fracddxexpbig(frac1(x - 1)^2big) text by L'Hôpital \
&= 2 lim_x to 1^+frac-3(x - 1)^-4-2expbig(frac1(x - 1)^2big)(x - 1)^-3 \
&= 3lim_x to 1^+frac(x - 1)^-1expbig(frac1(x - 1)^2big) quadtextagain, this has the form fracinftyinfty text so L'Hôpital applies \
&= 3 lim_x to 1^+fracfracddx(x - 1)^-1fracddxexpbig(frac1(x - 1)^2big) text by L'Hôpital \
&= 3 lim_x to 1^+frac-(x - 1)^-2-2expbig(frac1(x - 1)^2big)(x - 1)^-3 \
&= frac32 lim_x to 1^+fracx - 1expbig(frac1(x - 1)^2big) \
&= frac32 lim_x to 1^+ Big[(x - 1)expBig(-frac1(x - 1)^2Big)Big] \
&= frac32 Big[lim_x to 1^+ (x - 1)Big] Big[lim_x to 1^+ expBig(-frac1(x - 1)^2Big)Big] = frac32 times 0 times 0 = 0
endalign*
That was a long calculation but take my word: it can be repeated inductively to show that $limlimits_x to 1+fracd^nfdx^n = 0$ for all $n in mathbbZ_+!$ At all other points i.e. on $(-infty, -1) cup (-1, 1) cup (1, infty)$, $f$ is infinitely differentiable because exponentials and constant functions are infinitely differentiable.



Bonus Fact:



Both $fracd^n f(x)dx^n$ and $fracd^n g(x)dx^n$ also have the same value $0$ on $[-1, 1]$ for all positive integers $n$!



Edit:



A more elementary way of seeing $lim_x to 1^+fracdf(x)dx = 0$ is as follows:
beginalign*
lim_x to 1^+fracdf(x)dx &= 2lim_x to 1^+fracfrac1(x - 1)^3expbig(frac1(x - 1)^2big) \
&= 2lim_y to +inftyfracy^3expbig(y^2big) quadtext changing variables to y = frac1x - 1 \
&= 2lim_y to +inftyfracy^3sumlimits_n = 0^infty fracy^2nn! \
&= 2lim_y to +inftyfracy^31 + y^2 + fracy^42 + sumlimits_n = 3^infty fracy^2nn! \
&= 2lim_y to +inftyfrac1frac1y^3 + frac1y + fracy2 + sumlimits_n = 3^infty fracy^2n - 3n! quadtext dividing top and bottom by y^3 \
&= 2 times 0 = 0
endalign*
Essentially, the top term $1$, which remains constant, is a sitting duck. Meanwhile the bottom term blows up to infinity because in the bottom, only two terms ($frac1y^3$ and $frac1y$) go to zero as $y$ becomes large. The rest of the terms ($fracy2$ and $fracy^2n - 3n!$ for $n in mathbbZ_+ geq 3$) have at least a $y$ in the numerator and since no cancellation happens (all terms are positive), their sum becomes arbitrarily large and the overall ratio becomes arbitrarily small.






share|cite|improve this answer











$endgroup$












  • $begingroup$
    Well Done! Unfortunately, I am a high school student and never heard of L'Hôpitals Rule. EDIT: This function is actually amazing, never saw something like this before.
    $endgroup$
    – TVSuchty
    2 days ago







  • 2




    $begingroup$
    It basically says under certain conditions, $limlimits_x to a(f(x) / g(x)) = limlimits_x to a(fracd f(x)dx / fracd g(x)dx)$. en.wikipedia.org/wiki/L%27H%C3%B4pital%27s_rule
    $endgroup$
    – ZeroXLR
    2 days ago







  • 1




    $begingroup$
    I am stunned. Do you know of more complex solutions?
    $endgroup$
    – TVSuchty
    2 days ago






  • 1




    $begingroup$
    @TVSuchty what class shows you logic symbols before explaining L'Hôpitals rule?
    $endgroup$
    – uhhhhidk
    2 days ago






  • 1




    $begingroup$
    We can obtain $f^(n)(1)=0$ for all $n$ without l'Hopital's Rule by induction and some elementary considerations........+1
    $endgroup$
    – DanielWainfleet
    yesterday



















4












$begingroup$

Sure. In fact, there is a whole class of functions which not only exist, but are specifically made to do something that effectively implies exactly what you are looking for: they are called bump functions, and are defined as smooth (differentiable everywhere, arbitrarily many times) functions that have compact support, meaning (almost) that they are zero everywhere except on a compact set, which for the real numbers as domain basically means a closed, bounded (i.e. contained within an interval) subset thereof, such as a closed interval. This compact set where they are nonzero is called the "support". The trick is to exploit the "everywhere else zero"-ness, as that gives you what you're after.



Namely, any two different (i.e. not equal) bump functions with the same supporting interval $[a, b]$, will be smooth, zero on any interval outside this interval, and yet different, because they differ on such interval. More generally, given any two different bump functions, period, you just have to find an interval outside both of their support sets, which is always possible because they are both bounded.



A simple example of such a bump function is



$$mathrmbump: mathbbR rightarrow mathbbR, mathrmbump(x) := begincases e^-frac11 - x^2, mboxwhen $x$ is in $(-1, 1)$\ 0, mboxotherwise endcases$$



Then consider just $mathrmbump(x)$ and a nonzero multiple thereof, say, $mathrmbump2(x) := 2 cdot mathrmbump(x)$. We now have $mathrmbump(x) = mathrmbump2(x)$ when, say, $x in [10, 11]$, since they are both zero there. Yet, they are ostensibly not equal when $x$ is in $(-1, 1)$.



ADD: It appears to have been asked as to how one can do this without explicitly constructing the bump function. The above is just to show (not completely thoroughly) that bump functions exist. Indeed, we can do so as well. Let now $mathrmbump(x)$ be a general bump function. Let its support set be $mathrmsupp[mathrmbump]$. That is,



$$mathrmsupp[mathrmbump] := mathrmclleft( x in mathbbR : mathrmbump(x) ne 0 right)$$



(n.b. "cl" means to take the closure; basically this includes all "endpoints" of regions in which it is nonzero, even if it is zero at those endpoints - e.g. the support of the just-given-explicitly bump function is $[-1, 1]$, not $(-1, 1)$. This is a bit of technicality that was wrapped earlier when I said "almost" in "meaning (almost)" above.)



Since the support set is bounded and closed, it has a maximum and minimum (largest and smallest element): assign $M := mathrmmax mathrmsupp[mathrmbump]$. Now consider the interval $mathrmext ival := [M+1, M+2]$. If $x in mathrmext ival$, then it is clearly not in the support set, but rather to the right of it. Thus $mathrmbump(x) = 0$ there. Now set $mathrmbump2(x) := 2 cdot mathrmbump(x)$ as before (if you want even more generality, just replace $2$ with an arbitrary vertical rescaling coefficient $a$ that is not $0$ or $1$). Congrats, you now have two bump functions that are unequal but equal on the external interval $mathrmext ival$.






share|cite|improve this answer











$endgroup$








  • 1




    $begingroup$
    I see what you did there! Is there a case, where you can write an example without defining the function bit by bit, maybe just as term?
    $endgroup$
    – TVSuchty
    yesterday






  • 2




    $begingroup$
    @TVSuchty : Actually, yes. See what I just added. I believe this is what you're asking about.
    $endgroup$
    – The_Sympathizer
    yesterday






  • 1




    $begingroup$
    @TVSuchty : "I see what you did there!" yeah I just made the little bumpzzle perk up :) #mehhr.
    $endgroup$
    – The_Sympathizer
    yesterday











Your Answer








StackExchange.ready(function()
var channelOptions =
tags: "".split(" "),
id: "69"
;
initTagRenderer("".split(" "), "".split(" "), channelOptions);

StackExchange.using("externalEditor", function()
// Have to fire editor after snippets, if snippets enabled
if (StackExchange.settings.snippets.snippetsEnabled)
StackExchange.using("snippets", function()
createEditor();
);

else
createEditor();

);

function createEditor()
StackExchange.prepareEditor(
heartbeatType: 'answer',
autoActivateHeartbeat: false,
convertImagesToLinks: true,
noModals: true,
showLowRepImageUploadWarning: true,
reputationToPostImages: 10,
bindNavPrevention: true,
postfix: "",
imageUploader:
brandingHtml: "Powered by u003ca class="icon-imgur-white" href="https://imgur.com/"u003eu003c/au003e",
contentPolicyHtml: "User contributions licensed under u003ca href="https://creativecommons.org/licenses/by-sa/3.0/"u003ecc by-sa 3.0 with attribution requiredu003c/au003e u003ca href="https://stackoverflow.com/legal/content-policy"u003e(content policy)u003c/au003e",
allowUrls: true
,
noCode: true, onDemand: true,
discardSelector: ".discard-answer"
,immediatelyShowMarkdownHelp:true
);



);













draft saved

draft discarded


















StackExchange.ready(
function ()
StackExchange.openid.initPostLogin('.new-post-login', 'https%3a%2f%2fmath.stackexchange.com%2fquestions%2f3185630%2fare-there-continuous-functions-who-are-the-same-in-an-interval-but-differ-in-at%23new-answer', 'question_page');

);

Post as a guest















Required, but never shown

























2 Answers
2






active

oldest

votes








2 Answers
2






active

oldest

votes









active

oldest

votes






active

oldest

votes









16












$begingroup$

Define the real functions $f$ and $g$ thus:
$$
f(x) = begincases expBig(-frac1(x - 1)^2Big) &textif x > 1 \
0 &textif x in [-1, 1] \
expBig(-frac1(x + 1)^2Big) &textif x < -1
endcases
$$
and
$g(x) = 0$. $f$ and $g$ are both $0$ on $[-1, 1]$ but they differ in value everywhere else.



Obviously $g$ is continuously differentiable infinitely many times as it is a constant function. You can also check that $f$ is continuously differentiable infinitely many times at $x = -1$ and $x = 1$ by applying L'Hôpital's rule inductively (Edit: A more elementary demonstration is given below based on the exponential series). Checking this is a fine exercise in Real Analysis; you should try it. Here is a first taste of it:
beginalign*
lim_x to 1^+fracdf(x)dx &= limlimits_x to 1^+frac2expbig(- frac1(x - 1)^2big)(x - 1)^3 \
&= 2lim_x to 1^+fracfrac1(x - 1)^3expbig(frac1(x - 1)^2big) quadtextthis limit is of the form fracinftyinfty text so L'Hôpital applies \
&= 2 lim_x to 1^+fracfracddx(x - 1)^-3fracddxexpbig(frac1(x - 1)^2big) text by L'Hôpital \
&= 2 lim_x to 1^+frac-3(x - 1)^-4-2expbig(frac1(x - 1)^2big)(x - 1)^-3 \
&= 3lim_x to 1^+frac(x - 1)^-1expbig(frac1(x - 1)^2big) quadtextagain, this has the form fracinftyinfty text so L'Hôpital applies \
&= 3 lim_x to 1^+fracfracddx(x - 1)^-1fracddxexpbig(frac1(x - 1)^2big) text by L'Hôpital \
&= 3 lim_x to 1^+frac-(x - 1)^-2-2expbig(frac1(x - 1)^2big)(x - 1)^-3 \
&= frac32 lim_x to 1^+fracx - 1expbig(frac1(x - 1)^2big) \
&= frac32 lim_x to 1^+ Big[(x - 1)expBig(-frac1(x - 1)^2Big)Big] \
&= frac32 Big[lim_x to 1^+ (x - 1)Big] Big[lim_x to 1^+ expBig(-frac1(x - 1)^2Big)Big] = frac32 times 0 times 0 = 0
endalign*
That was a long calculation but take my word: it can be repeated inductively to show that $limlimits_x to 1+fracd^nfdx^n = 0$ for all $n in mathbbZ_+!$ At all other points i.e. on $(-infty, -1) cup (-1, 1) cup (1, infty)$, $f$ is infinitely differentiable because exponentials and constant functions are infinitely differentiable.



Bonus Fact:



Both $fracd^n f(x)dx^n$ and $fracd^n g(x)dx^n$ also have the same value $0$ on $[-1, 1]$ for all positive integers $n$!



Edit:



A more elementary way of seeing $lim_x to 1^+fracdf(x)dx = 0$ is as follows:
beginalign*
lim_x to 1^+fracdf(x)dx &= 2lim_x to 1^+fracfrac1(x - 1)^3expbig(frac1(x - 1)^2big) \
&= 2lim_y to +inftyfracy^3expbig(y^2big) quadtext changing variables to y = frac1x - 1 \
&= 2lim_y to +inftyfracy^3sumlimits_n = 0^infty fracy^2nn! \
&= 2lim_y to +inftyfracy^31 + y^2 + fracy^42 + sumlimits_n = 3^infty fracy^2nn! \
&= 2lim_y to +inftyfrac1frac1y^3 + frac1y + fracy2 + sumlimits_n = 3^infty fracy^2n - 3n! quadtext dividing top and bottom by y^3 \
&= 2 times 0 = 0
endalign*
Essentially, the top term $1$, which remains constant, is a sitting duck. Meanwhile the bottom term blows up to infinity because in the bottom, only two terms ($frac1y^3$ and $frac1y$) go to zero as $y$ becomes large. The rest of the terms ($fracy2$ and $fracy^2n - 3n!$ for $n in mathbbZ_+ geq 3$) have at least a $y$ in the numerator and since no cancellation happens (all terms are positive), their sum becomes arbitrarily large and the overall ratio becomes arbitrarily small.






share|cite|improve this answer











$endgroup$












  • $begingroup$
    Well Done! Unfortunately, I am a high school student and never heard of L'Hôpitals Rule. EDIT: This function is actually amazing, never saw something like this before.
    $endgroup$
    – TVSuchty
    2 days ago







  • 2




    $begingroup$
    It basically says under certain conditions, $limlimits_x to a(f(x) / g(x)) = limlimits_x to a(fracd f(x)dx / fracd g(x)dx)$. en.wikipedia.org/wiki/L%27H%C3%B4pital%27s_rule
    $endgroup$
    – ZeroXLR
    2 days ago







  • 1




    $begingroup$
    I am stunned. Do you know of more complex solutions?
    $endgroup$
    – TVSuchty
    2 days ago






  • 1




    $begingroup$
    @TVSuchty what class shows you logic symbols before explaining L'Hôpitals rule?
    $endgroup$
    – uhhhhidk
    2 days ago






  • 1




    $begingroup$
    We can obtain $f^(n)(1)=0$ for all $n$ without l'Hopital's Rule by induction and some elementary considerations........+1
    $endgroup$
    – DanielWainfleet
    yesterday
















16












$begingroup$

Define the real functions $f$ and $g$ thus:
$$
f(x) = begincases expBig(-frac1(x - 1)^2Big) &textif x > 1 \
0 &textif x in [-1, 1] \
expBig(-frac1(x + 1)^2Big) &textif x < -1
endcases
$$
and
$g(x) = 0$. $f$ and $g$ are both $0$ on $[-1, 1]$ but they differ in value everywhere else.



Obviously $g$ is continuously differentiable infinitely many times as it is a constant function. You can also check that $f$ is continuously differentiable infinitely many times at $x = -1$ and $x = 1$ by applying L'Hôpital's rule inductively (Edit: A more elementary demonstration is given below based on the exponential series). Checking this is a fine exercise in Real Analysis; you should try it. Here is a first taste of it:
beginalign*
lim_x to 1^+fracdf(x)dx &= limlimits_x to 1^+frac2expbig(- frac1(x - 1)^2big)(x - 1)^3 \
&= 2lim_x to 1^+fracfrac1(x - 1)^3expbig(frac1(x - 1)^2big) quadtextthis limit is of the form fracinftyinfty text so L'Hôpital applies \
&= 2 lim_x to 1^+fracfracddx(x - 1)^-3fracddxexpbig(frac1(x - 1)^2big) text by L'Hôpital \
&= 2 lim_x to 1^+frac-3(x - 1)^-4-2expbig(frac1(x - 1)^2big)(x - 1)^-3 \
&= 3lim_x to 1^+frac(x - 1)^-1expbig(frac1(x - 1)^2big) quadtextagain, this has the form fracinftyinfty text so L'Hôpital applies \
&= 3 lim_x to 1^+fracfracddx(x - 1)^-1fracddxexpbig(frac1(x - 1)^2big) text by L'Hôpital \
&= 3 lim_x to 1^+frac-(x - 1)^-2-2expbig(frac1(x - 1)^2big)(x - 1)^-3 \
&= frac32 lim_x to 1^+fracx - 1expbig(frac1(x - 1)^2big) \
&= frac32 lim_x to 1^+ Big[(x - 1)expBig(-frac1(x - 1)^2Big)Big] \
&= frac32 Big[lim_x to 1^+ (x - 1)Big] Big[lim_x to 1^+ expBig(-frac1(x - 1)^2Big)Big] = frac32 times 0 times 0 = 0
endalign*
That was a long calculation but take my word: it can be repeated inductively to show that $limlimits_x to 1+fracd^nfdx^n = 0$ for all $n in mathbbZ_+!$ At all other points i.e. on $(-infty, -1) cup (-1, 1) cup (1, infty)$, $f$ is infinitely differentiable because exponentials and constant functions are infinitely differentiable.



Bonus Fact:



Both $fracd^n f(x)dx^n$ and $fracd^n g(x)dx^n$ also have the same value $0$ on $[-1, 1]$ for all positive integers $n$!



Edit:



A more elementary way of seeing $lim_x to 1^+fracdf(x)dx = 0$ is as follows:
beginalign*
lim_x to 1^+fracdf(x)dx &= 2lim_x to 1^+fracfrac1(x - 1)^3expbig(frac1(x - 1)^2big) \
&= 2lim_y to +inftyfracy^3expbig(y^2big) quadtext changing variables to y = frac1x - 1 \
&= 2lim_y to +inftyfracy^3sumlimits_n = 0^infty fracy^2nn! \
&= 2lim_y to +inftyfracy^31 + y^2 + fracy^42 + sumlimits_n = 3^infty fracy^2nn! \
&= 2lim_y to +inftyfrac1frac1y^3 + frac1y + fracy2 + sumlimits_n = 3^infty fracy^2n - 3n! quadtext dividing top and bottom by y^3 \
&= 2 times 0 = 0
endalign*
Essentially, the top term $1$, which remains constant, is a sitting duck. Meanwhile the bottom term blows up to infinity because in the bottom, only two terms ($frac1y^3$ and $frac1y$) go to zero as $y$ becomes large. The rest of the terms ($fracy2$ and $fracy^2n - 3n!$ for $n in mathbbZ_+ geq 3$) have at least a $y$ in the numerator and since no cancellation happens (all terms are positive), their sum becomes arbitrarily large and the overall ratio becomes arbitrarily small.






share|cite|improve this answer











$endgroup$












  • $begingroup$
    Well Done! Unfortunately, I am a high school student and never heard of L'Hôpitals Rule. EDIT: This function is actually amazing, never saw something like this before.
    $endgroup$
    – TVSuchty
    2 days ago







  • 2




    $begingroup$
    It basically says under certain conditions, $limlimits_x to a(f(x) / g(x)) = limlimits_x to a(fracd f(x)dx / fracd g(x)dx)$. en.wikipedia.org/wiki/L%27H%C3%B4pital%27s_rule
    $endgroup$
    – ZeroXLR
    2 days ago







  • 1




    $begingroup$
    I am stunned. Do you know of more complex solutions?
    $endgroup$
    – TVSuchty
    2 days ago






  • 1




    $begingroup$
    @TVSuchty what class shows you logic symbols before explaining L'Hôpitals rule?
    $endgroup$
    – uhhhhidk
    2 days ago






  • 1




    $begingroup$
    We can obtain $f^(n)(1)=0$ for all $n$ without l'Hopital's Rule by induction and some elementary considerations........+1
    $endgroup$
    – DanielWainfleet
    yesterday














16












16








16





$begingroup$

Define the real functions $f$ and $g$ thus:
$$
f(x) = begincases expBig(-frac1(x - 1)^2Big) &textif x > 1 \
0 &textif x in [-1, 1] \
expBig(-frac1(x + 1)^2Big) &textif x < -1
endcases
$$
and
$g(x) = 0$. $f$ and $g$ are both $0$ on $[-1, 1]$ but they differ in value everywhere else.



Obviously $g$ is continuously differentiable infinitely many times as it is a constant function. You can also check that $f$ is continuously differentiable infinitely many times at $x = -1$ and $x = 1$ by applying L'Hôpital's rule inductively (Edit: A more elementary demonstration is given below based on the exponential series). Checking this is a fine exercise in Real Analysis; you should try it. Here is a first taste of it:
beginalign*
lim_x to 1^+fracdf(x)dx &= limlimits_x to 1^+frac2expbig(- frac1(x - 1)^2big)(x - 1)^3 \
&= 2lim_x to 1^+fracfrac1(x - 1)^3expbig(frac1(x - 1)^2big) quadtextthis limit is of the form fracinftyinfty text so L'Hôpital applies \
&= 2 lim_x to 1^+fracfracddx(x - 1)^-3fracddxexpbig(frac1(x - 1)^2big) text by L'Hôpital \
&= 2 lim_x to 1^+frac-3(x - 1)^-4-2expbig(frac1(x - 1)^2big)(x - 1)^-3 \
&= 3lim_x to 1^+frac(x - 1)^-1expbig(frac1(x - 1)^2big) quadtextagain, this has the form fracinftyinfty text so L'Hôpital applies \
&= 3 lim_x to 1^+fracfracddx(x - 1)^-1fracddxexpbig(frac1(x - 1)^2big) text by L'Hôpital \
&= 3 lim_x to 1^+frac-(x - 1)^-2-2expbig(frac1(x - 1)^2big)(x - 1)^-3 \
&= frac32 lim_x to 1^+fracx - 1expbig(frac1(x - 1)^2big) \
&= frac32 lim_x to 1^+ Big[(x - 1)expBig(-frac1(x - 1)^2Big)Big] \
&= frac32 Big[lim_x to 1^+ (x - 1)Big] Big[lim_x to 1^+ expBig(-frac1(x - 1)^2Big)Big] = frac32 times 0 times 0 = 0
endalign*
That was a long calculation but take my word: it can be repeated inductively to show that $limlimits_x to 1+fracd^nfdx^n = 0$ for all $n in mathbbZ_+!$ At all other points i.e. on $(-infty, -1) cup (-1, 1) cup (1, infty)$, $f$ is infinitely differentiable because exponentials and constant functions are infinitely differentiable.



Bonus Fact:



Both $fracd^n f(x)dx^n$ and $fracd^n g(x)dx^n$ also have the same value $0$ on $[-1, 1]$ for all positive integers $n$!



Edit:



A more elementary way of seeing $lim_x to 1^+fracdf(x)dx = 0$ is as follows:
beginalign*
lim_x to 1^+fracdf(x)dx &= 2lim_x to 1^+fracfrac1(x - 1)^3expbig(frac1(x - 1)^2big) \
&= 2lim_y to +inftyfracy^3expbig(y^2big) quadtext changing variables to y = frac1x - 1 \
&= 2lim_y to +inftyfracy^3sumlimits_n = 0^infty fracy^2nn! \
&= 2lim_y to +inftyfracy^31 + y^2 + fracy^42 + sumlimits_n = 3^infty fracy^2nn! \
&= 2lim_y to +inftyfrac1frac1y^3 + frac1y + fracy2 + sumlimits_n = 3^infty fracy^2n - 3n! quadtext dividing top and bottom by y^3 \
&= 2 times 0 = 0
endalign*
Essentially, the top term $1$, which remains constant, is a sitting duck. Meanwhile the bottom term blows up to infinity because in the bottom, only two terms ($frac1y^3$ and $frac1y$) go to zero as $y$ becomes large. The rest of the terms ($fracy2$ and $fracy^2n - 3n!$ for $n in mathbbZ_+ geq 3$) have at least a $y$ in the numerator and since no cancellation happens (all terms are positive), their sum becomes arbitrarily large and the overall ratio becomes arbitrarily small.






share|cite|improve this answer











$endgroup$



Define the real functions $f$ and $g$ thus:
$$
f(x) = begincases expBig(-frac1(x - 1)^2Big) &textif x > 1 \
0 &textif x in [-1, 1] \
expBig(-frac1(x + 1)^2Big) &textif x < -1
endcases
$$
and
$g(x) = 0$. $f$ and $g$ are both $0$ on $[-1, 1]$ but they differ in value everywhere else.



Obviously $g$ is continuously differentiable infinitely many times as it is a constant function. You can also check that $f$ is continuously differentiable infinitely many times at $x = -1$ and $x = 1$ by applying L'Hôpital's rule inductively (Edit: A more elementary demonstration is given below based on the exponential series). Checking this is a fine exercise in Real Analysis; you should try it. Here is a first taste of it:
beginalign*
lim_x to 1^+fracdf(x)dx &= limlimits_x to 1^+frac2expbig(- frac1(x - 1)^2big)(x - 1)^3 \
&= 2lim_x to 1^+fracfrac1(x - 1)^3expbig(frac1(x - 1)^2big) quadtextthis limit is of the form fracinftyinfty text so L'Hôpital applies \
&= 2 lim_x to 1^+fracfracddx(x - 1)^-3fracddxexpbig(frac1(x - 1)^2big) text by L'Hôpital \
&= 2 lim_x to 1^+frac-3(x - 1)^-4-2expbig(frac1(x - 1)^2big)(x - 1)^-3 \
&= 3lim_x to 1^+frac(x - 1)^-1expbig(frac1(x - 1)^2big) quadtextagain, this has the form fracinftyinfty text so L'Hôpital applies \
&= 3 lim_x to 1^+fracfracddx(x - 1)^-1fracddxexpbig(frac1(x - 1)^2big) text by L'Hôpital \
&= 3 lim_x to 1^+frac-(x - 1)^-2-2expbig(frac1(x - 1)^2big)(x - 1)^-3 \
&= frac32 lim_x to 1^+fracx - 1expbig(frac1(x - 1)^2big) \
&= frac32 lim_x to 1^+ Big[(x - 1)expBig(-frac1(x - 1)^2Big)Big] \
&= frac32 Big[lim_x to 1^+ (x - 1)Big] Big[lim_x to 1^+ expBig(-frac1(x - 1)^2Big)Big] = frac32 times 0 times 0 = 0
endalign*
That was a long calculation but take my word: it can be repeated inductively to show that $limlimits_x to 1+fracd^nfdx^n = 0$ for all $n in mathbbZ_+!$ At all other points i.e. on $(-infty, -1) cup (-1, 1) cup (1, infty)$, $f$ is infinitely differentiable because exponentials and constant functions are infinitely differentiable.



Bonus Fact:



Both $fracd^n f(x)dx^n$ and $fracd^n g(x)dx^n$ also have the same value $0$ on $[-1, 1]$ for all positive integers $n$!



Edit:



A more elementary way of seeing $lim_x to 1^+fracdf(x)dx = 0$ is as follows:
beginalign*
lim_x to 1^+fracdf(x)dx &= 2lim_x to 1^+fracfrac1(x - 1)^3expbig(frac1(x - 1)^2big) \
&= 2lim_y to +inftyfracy^3expbig(y^2big) quadtext changing variables to y = frac1x - 1 \
&= 2lim_y to +inftyfracy^3sumlimits_n = 0^infty fracy^2nn! \
&= 2lim_y to +inftyfracy^31 + y^2 + fracy^42 + sumlimits_n = 3^infty fracy^2nn! \
&= 2lim_y to +inftyfrac1frac1y^3 + frac1y + fracy2 + sumlimits_n = 3^infty fracy^2n - 3n! quadtext dividing top and bottom by y^3 \
&= 2 times 0 = 0
endalign*
Essentially, the top term $1$, which remains constant, is a sitting duck. Meanwhile the bottom term blows up to infinity because in the bottom, only two terms ($frac1y^3$ and $frac1y$) go to zero as $y$ becomes large. The rest of the terms ($fracy2$ and $fracy^2n - 3n!$ for $n in mathbbZ_+ geq 3$) have at least a $y$ in the numerator and since no cancellation happens (all terms are positive), their sum becomes arbitrarily large and the overall ratio becomes arbitrarily small.







share|cite|improve this answer














share|cite|improve this answer



share|cite|improve this answer








edited yesterday

























answered 2 days ago









ZeroXLRZeroXLR

1,657620




1,657620











  • $begingroup$
    Well Done! Unfortunately, I am a high school student and never heard of L'Hôpitals Rule. EDIT: This function is actually amazing, never saw something like this before.
    $endgroup$
    – TVSuchty
    2 days ago







  • 2




    $begingroup$
    It basically says under certain conditions, $limlimits_x to a(f(x) / g(x)) = limlimits_x to a(fracd f(x)dx / fracd g(x)dx)$. en.wikipedia.org/wiki/L%27H%C3%B4pital%27s_rule
    $endgroup$
    – ZeroXLR
    2 days ago







  • 1




    $begingroup$
    I am stunned. Do you know of more complex solutions?
    $endgroup$
    – TVSuchty
    2 days ago






  • 1




    $begingroup$
    @TVSuchty what class shows you logic symbols before explaining L'Hôpitals rule?
    $endgroup$
    – uhhhhidk
    2 days ago






  • 1




    $begingroup$
    We can obtain $f^(n)(1)=0$ for all $n$ without l'Hopital's Rule by induction and some elementary considerations........+1
    $endgroup$
    – DanielWainfleet
    yesterday

















  • $begingroup$
    Well Done! Unfortunately, I am a high school student and never heard of L'Hôpitals Rule. EDIT: This function is actually amazing, never saw something like this before.
    $endgroup$
    – TVSuchty
    2 days ago







  • 2




    $begingroup$
    It basically says under certain conditions, $limlimits_x to a(f(x) / g(x)) = limlimits_x to a(fracd f(x)dx / fracd g(x)dx)$. en.wikipedia.org/wiki/L%27H%C3%B4pital%27s_rule
    $endgroup$
    – ZeroXLR
    2 days ago







  • 1




    $begingroup$
    I am stunned. Do you know of more complex solutions?
    $endgroup$
    – TVSuchty
    2 days ago






  • 1




    $begingroup$
    @TVSuchty what class shows you logic symbols before explaining L'Hôpitals rule?
    $endgroup$
    – uhhhhidk
    2 days ago






  • 1




    $begingroup$
    We can obtain $f^(n)(1)=0$ for all $n$ without l'Hopital's Rule by induction and some elementary considerations........+1
    $endgroup$
    – DanielWainfleet
    yesterday
















$begingroup$
Well Done! Unfortunately, I am a high school student and never heard of L'Hôpitals Rule. EDIT: This function is actually amazing, never saw something like this before.
$endgroup$
– TVSuchty
2 days ago





$begingroup$
Well Done! Unfortunately, I am a high school student and never heard of L'Hôpitals Rule. EDIT: This function is actually amazing, never saw something like this before.
$endgroup$
– TVSuchty
2 days ago





2




2




$begingroup$
It basically says under certain conditions, $limlimits_x to a(f(x) / g(x)) = limlimits_x to a(fracd f(x)dx / fracd g(x)dx)$. en.wikipedia.org/wiki/L%27H%C3%B4pital%27s_rule
$endgroup$
– ZeroXLR
2 days ago





$begingroup$
It basically says under certain conditions, $limlimits_x to a(f(x) / g(x)) = limlimits_x to a(fracd f(x)dx / fracd g(x)dx)$. en.wikipedia.org/wiki/L%27H%C3%B4pital%27s_rule
$endgroup$
– ZeroXLR
2 days ago





1




1




$begingroup$
I am stunned. Do you know of more complex solutions?
$endgroup$
– TVSuchty
2 days ago




$begingroup$
I am stunned. Do you know of more complex solutions?
$endgroup$
– TVSuchty
2 days ago




1




1




$begingroup$
@TVSuchty what class shows you logic symbols before explaining L'Hôpitals rule?
$endgroup$
– uhhhhidk
2 days ago




$begingroup$
@TVSuchty what class shows you logic symbols before explaining L'Hôpitals rule?
$endgroup$
– uhhhhidk
2 days ago




1




1




$begingroup$
We can obtain $f^(n)(1)=0$ for all $n$ without l'Hopital's Rule by induction and some elementary considerations........+1
$endgroup$
– DanielWainfleet
yesterday





$begingroup$
We can obtain $f^(n)(1)=0$ for all $n$ without l'Hopital's Rule by induction and some elementary considerations........+1
$endgroup$
– DanielWainfleet
yesterday












4












$begingroup$

Sure. In fact, there is a whole class of functions which not only exist, but are specifically made to do something that effectively implies exactly what you are looking for: they are called bump functions, and are defined as smooth (differentiable everywhere, arbitrarily many times) functions that have compact support, meaning (almost) that they are zero everywhere except on a compact set, which for the real numbers as domain basically means a closed, bounded (i.e. contained within an interval) subset thereof, such as a closed interval. This compact set where they are nonzero is called the "support". The trick is to exploit the "everywhere else zero"-ness, as that gives you what you're after.



Namely, any two different (i.e. not equal) bump functions with the same supporting interval $[a, b]$, will be smooth, zero on any interval outside this interval, and yet different, because they differ on such interval. More generally, given any two different bump functions, period, you just have to find an interval outside both of their support sets, which is always possible because they are both bounded.



A simple example of such a bump function is



$$mathrmbump: mathbbR rightarrow mathbbR, mathrmbump(x) := begincases e^-frac11 - x^2, mboxwhen $x$ is in $(-1, 1)$\ 0, mboxotherwise endcases$$



Then consider just $mathrmbump(x)$ and a nonzero multiple thereof, say, $mathrmbump2(x) := 2 cdot mathrmbump(x)$. We now have $mathrmbump(x) = mathrmbump2(x)$ when, say, $x in [10, 11]$, since they are both zero there. Yet, they are ostensibly not equal when $x$ is in $(-1, 1)$.



ADD: It appears to have been asked as to how one can do this without explicitly constructing the bump function. The above is just to show (not completely thoroughly) that bump functions exist. Indeed, we can do so as well. Let now $mathrmbump(x)$ be a general bump function. Let its support set be $mathrmsupp[mathrmbump]$. That is,



$$mathrmsupp[mathrmbump] := mathrmclleft( x in mathbbR : mathrmbump(x) ne 0 right)$$



(n.b. "cl" means to take the closure; basically this includes all "endpoints" of regions in which it is nonzero, even if it is zero at those endpoints - e.g. the support of the just-given-explicitly bump function is $[-1, 1]$, not $(-1, 1)$. This is a bit of technicality that was wrapped earlier when I said "almost" in "meaning (almost)" above.)



Since the support set is bounded and closed, it has a maximum and minimum (largest and smallest element): assign $M := mathrmmax mathrmsupp[mathrmbump]$. Now consider the interval $mathrmext ival := [M+1, M+2]$. If $x in mathrmext ival$, then it is clearly not in the support set, but rather to the right of it. Thus $mathrmbump(x) = 0$ there. Now set $mathrmbump2(x) := 2 cdot mathrmbump(x)$ as before (if you want even more generality, just replace $2$ with an arbitrary vertical rescaling coefficient $a$ that is not $0$ or $1$). Congrats, you now have two bump functions that are unequal but equal on the external interval $mathrmext ival$.






share|cite|improve this answer











$endgroup$








  • 1




    $begingroup$
    I see what you did there! Is there a case, where you can write an example without defining the function bit by bit, maybe just as term?
    $endgroup$
    – TVSuchty
    yesterday






  • 2




    $begingroup$
    @TVSuchty : Actually, yes. See what I just added. I believe this is what you're asking about.
    $endgroup$
    – The_Sympathizer
    yesterday






  • 1




    $begingroup$
    @TVSuchty : "I see what you did there!" yeah I just made the little bumpzzle perk up :) #mehhr.
    $endgroup$
    – The_Sympathizer
    yesterday















4












$begingroup$

Sure. In fact, there is a whole class of functions which not only exist, but are specifically made to do something that effectively implies exactly what you are looking for: they are called bump functions, and are defined as smooth (differentiable everywhere, arbitrarily many times) functions that have compact support, meaning (almost) that they are zero everywhere except on a compact set, which for the real numbers as domain basically means a closed, bounded (i.e. contained within an interval) subset thereof, such as a closed interval. This compact set where they are nonzero is called the "support". The trick is to exploit the "everywhere else zero"-ness, as that gives you what you're after.



Namely, any two different (i.e. not equal) bump functions with the same supporting interval $[a, b]$, will be smooth, zero on any interval outside this interval, and yet different, because they differ on such interval. More generally, given any two different bump functions, period, you just have to find an interval outside both of their support sets, which is always possible because they are both bounded.



A simple example of such a bump function is



$$mathrmbump: mathbbR rightarrow mathbbR, mathrmbump(x) := begincases e^-frac11 - x^2, mboxwhen $x$ is in $(-1, 1)$\ 0, mboxotherwise endcases$$



Then consider just $mathrmbump(x)$ and a nonzero multiple thereof, say, $mathrmbump2(x) := 2 cdot mathrmbump(x)$. We now have $mathrmbump(x) = mathrmbump2(x)$ when, say, $x in [10, 11]$, since they are both zero there. Yet, they are ostensibly not equal when $x$ is in $(-1, 1)$.



ADD: It appears to have been asked as to how one can do this without explicitly constructing the bump function. The above is just to show (not completely thoroughly) that bump functions exist. Indeed, we can do so as well. Let now $mathrmbump(x)$ be a general bump function. Let its support set be $mathrmsupp[mathrmbump]$. That is,



$$mathrmsupp[mathrmbump] := mathrmclleft( x in mathbbR : mathrmbump(x) ne 0 right)$$



(n.b. "cl" means to take the closure; basically this includes all "endpoints" of regions in which it is nonzero, even if it is zero at those endpoints - e.g. the support of the just-given-explicitly bump function is $[-1, 1]$, not $(-1, 1)$. This is a bit of technicality that was wrapped earlier when I said "almost" in "meaning (almost)" above.)



Since the support set is bounded and closed, it has a maximum and minimum (largest and smallest element): assign $M := mathrmmax mathrmsupp[mathrmbump]$. Now consider the interval $mathrmext ival := [M+1, M+2]$. If $x in mathrmext ival$, then it is clearly not in the support set, but rather to the right of it. Thus $mathrmbump(x) = 0$ there. Now set $mathrmbump2(x) := 2 cdot mathrmbump(x)$ as before (if you want even more generality, just replace $2$ with an arbitrary vertical rescaling coefficient $a$ that is not $0$ or $1$). Congrats, you now have two bump functions that are unequal but equal on the external interval $mathrmext ival$.






share|cite|improve this answer











$endgroup$








  • 1




    $begingroup$
    I see what you did there! Is there a case, where you can write an example without defining the function bit by bit, maybe just as term?
    $endgroup$
    – TVSuchty
    yesterday






  • 2




    $begingroup$
    @TVSuchty : Actually, yes. See what I just added. I believe this is what you're asking about.
    $endgroup$
    – The_Sympathizer
    yesterday






  • 1




    $begingroup$
    @TVSuchty : "I see what you did there!" yeah I just made the little bumpzzle perk up :) #mehhr.
    $endgroup$
    – The_Sympathizer
    yesterday













4












4








4





$begingroup$

Sure. In fact, there is a whole class of functions which not only exist, but are specifically made to do something that effectively implies exactly what you are looking for: they are called bump functions, and are defined as smooth (differentiable everywhere, arbitrarily many times) functions that have compact support, meaning (almost) that they are zero everywhere except on a compact set, which for the real numbers as domain basically means a closed, bounded (i.e. contained within an interval) subset thereof, such as a closed interval. This compact set where they are nonzero is called the "support". The trick is to exploit the "everywhere else zero"-ness, as that gives you what you're after.



Namely, any two different (i.e. not equal) bump functions with the same supporting interval $[a, b]$, will be smooth, zero on any interval outside this interval, and yet different, because they differ on such interval. More generally, given any two different bump functions, period, you just have to find an interval outside both of their support sets, which is always possible because they are both bounded.



A simple example of such a bump function is



$$mathrmbump: mathbbR rightarrow mathbbR, mathrmbump(x) := begincases e^-frac11 - x^2, mboxwhen $x$ is in $(-1, 1)$\ 0, mboxotherwise endcases$$



Then consider just $mathrmbump(x)$ and a nonzero multiple thereof, say, $mathrmbump2(x) := 2 cdot mathrmbump(x)$. We now have $mathrmbump(x) = mathrmbump2(x)$ when, say, $x in [10, 11]$, since they are both zero there. Yet, they are ostensibly not equal when $x$ is in $(-1, 1)$.



ADD: It appears to have been asked as to how one can do this without explicitly constructing the bump function. The above is just to show (not completely thoroughly) that bump functions exist. Indeed, we can do so as well. Let now $mathrmbump(x)$ be a general bump function. Let its support set be $mathrmsupp[mathrmbump]$. That is,



$$mathrmsupp[mathrmbump] := mathrmclleft( x in mathbbR : mathrmbump(x) ne 0 right)$$



(n.b. "cl" means to take the closure; basically this includes all "endpoints" of regions in which it is nonzero, even if it is zero at those endpoints - e.g. the support of the just-given-explicitly bump function is $[-1, 1]$, not $(-1, 1)$. This is a bit of technicality that was wrapped earlier when I said "almost" in "meaning (almost)" above.)



Since the support set is bounded and closed, it has a maximum and minimum (largest and smallest element): assign $M := mathrmmax mathrmsupp[mathrmbump]$. Now consider the interval $mathrmext ival := [M+1, M+2]$. If $x in mathrmext ival$, then it is clearly not in the support set, but rather to the right of it. Thus $mathrmbump(x) = 0$ there. Now set $mathrmbump2(x) := 2 cdot mathrmbump(x)$ as before (if you want even more generality, just replace $2$ with an arbitrary vertical rescaling coefficient $a$ that is not $0$ or $1$). Congrats, you now have two bump functions that are unequal but equal on the external interval $mathrmext ival$.






share|cite|improve this answer











$endgroup$



Sure. In fact, there is a whole class of functions which not only exist, but are specifically made to do something that effectively implies exactly what you are looking for: they are called bump functions, and are defined as smooth (differentiable everywhere, arbitrarily many times) functions that have compact support, meaning (almost) that they are zero everywhere except on a compact set, which for the real numbers as domain basically means a closed, bounded (i.e. contained within an interval) subset thereof, such as a closed interval. This compact set where they are nonzero is called the "support". The trick is to exploit the "everywhere else zero"-ness, as that gives you what you're after.



Namely, any two different (i.e. not equal) bump functions with the same supporting interval $[a, b]$, will be smooth, zero on any interval outside this interval, and yet different, because they differ on such interval. More generally, given any two different bump functions, period, you just have to find an interval outside both of their support sets, which is always possible because they are both bounded.



A simple example of such a bump function is



$$mathrmbump: mathbbR rightarrow mathbbR, mathrmbump(x) := begincases e^-frac11 - x^2, mboxwhen $x$ is in $(-1, 1)$\ 0, mboxotherwise endcases$$



Then consider just $mathrmbump(x)$ and a nonzero multiple thereof, say, $mathrmbump2(x) := 2 cdot mathrmbump(x)$. We now have $mathrmbump(x) = mathrmbump2(x)$ when, say, $x in [10, 11]$, since they are both zero there. Yet, they are ostensibly not equal when $x$ is in $(-1, 1)$.



ADD: It appears to have been asked as to how one can do this without explicitly constructing the bump function. The above is just to show (not completely thoroughly) that bump functions exist. Indeed, we can do so as well. Let now $mathrmbump(x)$ be a general bump function. Let its support set be $mathrmsupp[mathrmbump]$. That is,



$$mathrmsupp[mathrmbump] := mathrmclleft( x in mathbbR : mathrmbump(x) ne 0 right)$$



(n.b. "cl" means to take the closure; basically this includes all "endpoints" of regions in which it is nonzero, even if it is zero at those endpoints - e.g. the support of the just-given-explicitly bump function is $[-1, 1]$, not $(-1, 1)$. This is a bit of technicality that was wrapped earlier when I said "almost" in "meaning (almost)" above.)



Since the support set is bounded and closed, it has a maximum and minimum (largest and smallest element): assign $M := mathrmmax mathrmsupp[mathrmbump]$. Now consider the interval $mathrmext ival := [M+1, M+2]$. If $x in mathrmext ival$, then it is clearly not in the support set, but rather to the right of it. Thus $mathrmbump(x) = 0$ there. Now set $mathrmbump2(x) := 2 cdot mathrmbump(x)$ as before (if you want even more generality, just replace $2$ with an arbitrary vertical rescaling coefficient $a$ that is not $0$ or $1$). Congrats, you now have two bump functions that are unequal but equal on the external interval $mathrmext ival$.







share|cite|improve this answer














share|cite|improve this answer



share|cite|improve this answer








edited yesterday

























answered yesterday









The_SympathizerThe_Sympathizer

7,8752246




7,8752246







  • 1




    $begingroup$
    I see what you did there! Is there a case, where you can write an example without defining the function bit by bit, maybe just as term?
    $endgroup$
    – TVSuchty
    yesterday






  • 2




    $begingroup$
    @TVSuchty : Actually, yes. See what I just added. I believe this is what you're asking about.
    $endgroup$
    – The_Sympathizer
    yesterday






  • 1




    $begingroup$
    @TVSuchty : "I see what you did there!" yeah I just made the little bumpzzle perk up :) #mehhr.
    $endgroup$
    – The_Sympathizer
    yesterday












  • 1




    $begingroup$
    I see what you did there! Is there a case, where you can write an example without defining the function bit by bit, maybe just as term?
    $endgroup$
    – TVSuchty
    yesterday






  • 2




    $begingroup$
    @TVSuchty : Actually, yes. See what I just added. I believe this is what you're asking about.
    $endgroup$
    – The_Sympathizer
    yesterday






  • 1




    $begingroup$
    @TVSuchty : "I see what you did there!" yeah I just made the little bumpzzle perk up :) #mehhr.
    $endgroup$
    – The_Sympathizer
    yesterday







1




1




$begingroup$
I see what you did there! Is there a case, where you can write an example without defining the function bit by bit, maybe just as term?
$endgroup$
– TVSuchty
yesterday




$begingroup$
I see what you did there! Is there a case, where you can write an example without defining the function bit by bit, maybe just as term?
$endgroup$
– TVSuchty
yesterday




2




2




$begingroup$
@TVSuchty : Actually, yes. See what I just added. I believe this is what you're asking about.
$endgroup$
– The_Sympathizer
yesterday




$begingroup$
@TVSuchty : Actually, yes. See what I just added. I believe this is what you're asking about.
$endgroup$
– The_Sympathizer
yesterday




1




1




$begingroup$
@TVSuchty : "I see what you did there!" yeah I just made the little bumpzzle perk up :) #mehhr.
$endgroup$
– The_Sympathizer
yesterday




$begingroup$
@TVSuchty : "I see what you did there!" yeah I just made the little bumpzzle perk up :) #mehhr.
$endgroup$
– The_Sympathizer
yesterday

















draft saved

draft discarded
















































Thanks for contributing an answer to Mathematics Stack Exchange!


  • Please be sure to answer the question. Provide details and share your research!

But avoid


  • Asking for help, clarification, or responding to other answers.

  • Making statements based on opinion; back them up with references or personal experience.

Use MathJax to format equations. MathJax reference.


To learn more, see our tips on writing great answers.




draft saved


draft discarded














StackExchange.ready(
function ()
StackExchange.openid.initPostLogin('.new-post-login', 'https%3a%2f%2fmath.stackexchange.com%2fquestions%2f3185630%2fare-there-continuous-functions-who-are-the-same-in-an-interval-but-differ-in-at%23new-answer', 'question_page');

);

Post as a guest















Required, but never shown





















































Required, but never shown














Required, but never shown












Required, but never shown







Required, but never shown

































Required, but never shown














Required, but never shown












Required, but never shown







Required, but never shown







Popular posts from this blog

Get product attribute by attribute group code in magento 2get product attribute by product attribute group in magento 2Magento 2 Log Bundle Product Data in List Page?How to get all product attribute of a attribute group of Default attribute set?Magento 2.1 Create a filter in the product grid by new attributeMagento 2 : Get Product Attribute values By GroupMagento 2 How to get all existing values for one attributeMagento 2 get custom attribute of a single product inside a pluginMagento 2.3 How to get all the Multi Source Inventory (MSI) locations collection in custom module?Magento2: how to develop rest API to get new productsGet product attribute by attribute group code ( [attribute_group_code] ) in magento 2

Category:9 (number) SubcategoriesMedia in category "9 (number)"Navigation menuUpload mediaGND ID: 4485639-8Library of Congress authority ID: sh85091979ReasonatorScholiaStatistics

Magento 2.3: How do i solve this, Not registered handle, on custom form?How can i rewrite TierPrice Block in Magento2magento 2 captcha not rendering if I override layout xmlmain.CRITICAL: Plugin class doesn't existMagento 2 : Problem while adding custom button order view page?Magento 2.2.5: Overriding Admin Controller sales/orderMagento 2.2.5: Add, Update and Delete existing products Custom OptionsMagento 2.3 : File Upload issue in UI Component FormMagento2 Not registered handleHow to configured Form Builder Js in my custom magento 2.3.0 module?Magento 2.3. How to create image upload field in an admin form